Last visit was: 25 Apr 2024, 05:10 It is currently 25 Apr 2024, 05:10

Close
GMAT Club Daily Prep
Thank you for using the timer - this advanced tool can estimate your performance and suggest more practice questions. We have subscribed you to Daily Prep Questions via email.

Customized
for You

we will pick new questions that match your level based on your Timer History

Track
Your Progress

every week, we’ll send you an estimated GMAT score based on your performance

Practice
Pays

we will pick new questions that match your level based on your Timer History
Not interested in getting valuable practice questions and articles delivered to your email? No problem, unsubscribe here.
Close
Request Expert Reply
Confirm Cancel
SORT BY:
Date
Tags:
Show Tags
Hide Tags
Board of Directors
Joined: 01 Sep 2010
Posts: 4384
Own Kudos [?]: 32875 [237]
Given Kudos: 4455
Send PM
Most Helpful Reply
Math Expert
Joined: 02 Sep 2009
Posts: 92912
Own Kudos [?]: 618923 [89]
Given Kudos: 81595
Send PM
Tutor
Joined: 16 Oct 2010
Posts: 14822
Own Kudos [?]: 64909 [13]
Given Kudos: 426
Location: Pune, India
Send PM
General Discussion
User avatar
Intern
Intern
Joined: 07 Jan 2013
Posts: 29
Own Kudos [?]: 161 [2]
Given Kudos: 23
Location: India
Concentration: Finance, Strategy
GMAT 1: 570 Q46 V23
GMAT 2: 710 Q49 V38
GPA: 2.9
WE:Information Technology (Computer Software)
Send PM
Re: In the correctly worked addition problem above, A, B, C, D [#permalink]
2
Kudos
Bunuel wrote:
In the correctly worked addition problem above, A, B, C, D, E, F, and G are distinct digits. What is the sum of E, F, and G ?

AD
BD
CD
---
EFG

Notice that E can be only 1 or 2 (no sum of 3 two-digit numbers can give number more than 297).

(1) A, B, and C are consecutive odd integers. 3 cases are possible:

(i) A, B, and C are 1, 3, and 5 (it doesn't matter which is which) --> 1+3+5=9 then E (hundreds digit) can only be 1, which is not possible since we are told that the digits are distinct and we already have 1 (A, B, or C);

(ii) A, B, and C are 3, 5, and 7 (it doesn't matter which is which) --> 3+5+7=15 then E (hundreds digit) can only be 1. So, D can be 0, 2, 4, 6, 8, or 9. After trial and error we can get that only D=4 will give all distinct digits:
34
54
74
---
162

E+F+G=9.

(iii) A, B, and C are 5, 7, and 9 (it doesn't matter which is which) --> 5+7+9=21 then E (hundreds digit) can only be 2. So, D can be 1, 3, 4, 6, or 8. After trial and error we can get that only D=8 will give all distinct digits:
58
78
98
---
234

E+F+G=9.

So, as you can see in both valid cases (ii and iii) the sum of E, F, and G is 9. Sufficient.

(2) E = 2. After some trial and error you can find that several numbers can be found which will give different values for the sum of E, F, and G, for example: 58+78+98=234 and 38+78+98=214. Not sufficient.

Answer: A.

P.S. Though not very hard this question is not likely to appear on the GMAT because of long and boring math.


Hi Bunuel ,

Just curious to know why A=1,B=3,C=5 and sum = 9 and the resulting number 96 cannot be the case. It says any digits so why EFG =096 cant be a possibility??
Math Expert
Joined: 02 Sep 2009
Posts: 92912
Own Kudos [?]: 618923 [1]
Given Kudos: 81595
Send PM
Re: In the correctly worked addition problem above, A, B, C, D [#permalink]
1
Bookmarks
Expert Reply
adg142000 wrote:
Bunuel wrote:
In the correctly worked addition problem above, A, B, C, D, E, F, and G are distinct digits. What is the sum of E, F, and G ?

AD
BD
CD
---
EFG

Notice that E can be only 1 or 2 (no sum of 3 two-digit numbers can give number more than 297).

(1) A, B, and C are consecutive odd integers. 3 cases are possible:

(i) A, B, and C are 1, 3, and 5 (it doesn't matter which is which) --> 1+3+5=9 then E (hundreds digit) can only be 1, which is not possible since we are told that the digits are distinct and we already have 1 (A, B, or C);

(ii) A, B, and C are 3, 5, and 7 (it doesn't matter which is which) --> 3+5+7=15 then E (hundreds digit) can only be 1. So, D can be 0, 2, 4, 6, 8, or 9. After trial and error we can get that only D=4 will give all distinct digits:
34
54
74
---
162

E+F+G=9.

(iii) A, B, and C are 5, 7, and 9 (it doesn't matter which is which) --> 5+7+9=21 then E (hundreds digit) can only be 2. So, D can be 1, 3, 4, 6, or 8. After trial and error we can get that only D=8 will give all distinct digits:
58
78
98
---
234

E+F+G=9.

So, as you can see in both valid cases (ii and iii) the sum of E, F, and G is 9. Sufficient.

(2) E = 2. After some trial and error you can find that several numbers can be found which will give different values for the sum of E, F, and G, for example: 58+78+98=234 and 38+78+98=214. Not sufficient.

Answer: A.

P.S. Though not very hard this question is not likely to appear on the GMAT because of long and boring math.


Hi Bunuel ,

Just curious to know why A=1,B=3,C=5 and sum = 9 and the resulting number 96 cannot be the case. It says any digits so why EFG =096 cant be a possibility??


From the stem we can assume that E is not 0.
Tutor
Joined: 16 Oct 2010
Posts: 14822
Own Kudos [?]: 64909 [4]
Given Kudos: 426
Location: Pune, India
Send PM
Re: In the correctly worked addition problem above, A, B, C, D [#permalink]
4
Kudos
Expert Reply
adg142000 wrote:

Hi Bunuel ,

Just curious to know why A=1,B=3,C=5 and sum = 9 and the resulting number 96 cannot be the case. It says any digits so why EFG =096 cant be a possibility??


To add to what Bunuel said, the question stem tells us that when we add three 2 digit numbers, we get a 3 digit number. Had you obtained a 2 digit number as the sum, you would have wirtten the addition as

AD
BD
CD
---
FG

and not EFG.
avatar
Intern
Intern
Joined: 12 Jan 2015
Posts: 1
Own Kudos [?]: [0]
Given Kudos: 0
Send PM
Re: In the correctly worked addition problem above, A, B, C, D [#permalink]
Hi, Why cant it be AD =56; BD =76; CD =96 and EFG =218

In this case the sum is E+F+G = 11 only.
Math Expert
Joined: 02 Sep 2009
Posts: 92912
Own Kudos [?]: 618923 [0]
Given Kudos: 81595
Send PM
Re: In the correctly worked addition problem above, A, B, C, D [#permalink]
Expert Reply
karthikv606 wrote:
Hi, Why cant it be AD =56; BD =76; CD =96 and EFG =218

In this case the sum is E+F+G = 11 only.


56 + 76 + 96 = 228, not 218 and in this case E and F are not distinct.
Intern
Intern
Joined: 27 Aug 2015
Posts: 5
Own Kudos [?]: 3 [0]
Given Kudos: 27
Send PM
Re: In the correctly worked addition problem above, A, B, C, D [#permalink]
Bunuel wrote:
In the correctly worked addition problem above, A, B, C, D, E, F, and G are distinct digits. What is the sum of E, F, and G ?

AD
BD
CD
---
EFG

Notice that E can be only 1 or 2 (no sum of 3 two-digit numbers can give number more than 297).

(1) A, B, and C are consecutive odd integers. 3 cases are possible:

(i) A, B, and C are 1, 3, and 5 (it doesn't matter which is which) --> 1+3+5=9 then E (hundreds digit) can only be 1, which is not possible since we are told that the digits are distinct and we already have 1 (A, B, or C);

(ii) A, B, and C are 3, 5, and 7 (it doesn't matter which is which) --> 3+5+7=15 then E (hundreds digit) can only be 1. So, D can be 0, 2, 4, 6, 8, or 9. After trial and error we can get that only D=4 will give all distinct digits:
34
54
74
---
162

E+F+G=9.

(iii) A, B, and C are 5, 7, and 9 (it doesn't matter which is which) --> 5+7+9=21 then E (hundreds digit) can only be 2. So, D can be 1, 3, 4, 6, or 8. After trial and error we can get that only D=8 will give all distinct digits:
58
78
98
---
234

E+F+G=9.

So, as you can see in both valid cases (ii and iii) the sum of E, F, and G is 9. Sufficient.

(2) E = 2. After some trial and error you can find that several numbers can be found which will give different values for the sum of E, F, and G, for example: 58+78+98=234 and 38+78+98=214. Not sufficient.

Answer: A.

P.S. Though not very hard this question is not likely to appear on the GMAT because of long and boring math.


Hi Bunuel
In first statement ,Case 1,why have we assumed that E can only be =1
Math Expert
Joined: 02 Sep 2009
Posts: 92912
Own Kudos [?]: 618923 [0]
Given Kudos: 81595
Send PM
Re: In the correctly worked addition problem above, A, B, C, D [#permalink]
Expert Reply
ankit589 wrote:
Bunuel wrote:
In the correctly worked addition problem above, A, B, C, D, E, F, and G are distinct digits. What is the sum of E, F, and G ?

AD
BD
CD
---
EFG

Notice that E can be only 1 or 2 (no sum of 3 two-digit numbers can give number more than 297).

(1) A, B, and C are consecutive odd integers. 3 cases are possible:

(i) A, B, and C are 1, 3, and 5 (it doesn't matter which is which) --> 1+3+5=9 then E (hundreds digit) can only be 1, which is not possible since we are told that the digits are distinct and we already have 1 (A, B, or C);

(ii) A, B, and C are 3, 5, and 7 (it doesn't matter which is which) --> 3+5+7=15 then E (hundreds digit) can only be 1. So, D can be 0, 2, 4, 6, 8, or 9. After trial and error we can get that only D=4 will give all distinct digits:
34
54
74
---
162

E+F+G=9.

(iii) A, B, and C are 5, 7, and 9 (it doesn't matter which is which) --> 5+7+9=21 then E (hundreds digit) can only be 2. So, D can be 1, 3, 4, 6, or 8. After trial and error we can get that only D=8 will give all distinct digits:
58
78
98
---
234

E+F+G=9.

So, as you can see in both valid cases (ii and iii) the sum of E, F, and G is 9. Sufficient.

(2) E = 2. After some trial and error you can find that several numbers can be found which will give different values for the sum of E, F, and G, for example: 58+78+98=234 and 38+78+98=214. Not sufficient.

Answer: A.

P.S. Though not very hard this question is not likely to appear on the GMAT because of long and boring math.


Hi Bunuel
In first statement ,Case 1,why have we assumed that E can only be =1


If A, B, and C are 1, 3, and 5, then the sum is less than 200.
Manager
Manager
Joined: 07 May 2020
Posts: 67
Own Kudos [?]: 24 [0]
Given Kudos: 11
GMAT 1: 650 Q50 V28
GMAT 2: 720 Q50 V38 (Online)
Send PM
Re: In the correctly worked addition problem above, A, B, C, D [#permalink]
Hello Bunuel

I had a scenario like this

3 4
5 4
+ 7 4
------------
1 6 2

and the other can be

5 8
7 8
+ 9 8
-----------
2 3 4

Then how can A be sufficient?
Math Expert
Joined: 02 Sep 2009
Posts: 92912
Own Kudos [?]: 618923 [1]
Given Kudos: 81595
Send PM
Re: In the correctly worked addition problem above, A, B, C, D [#permalink]
1
Bookmarks
Expert Reply
41396302717 wrote:
Hello Bunuel

I had a scenario like this

3 4
5 4
+ 7 4
------------
1 6 2

and the other can be

5 8
7 8
+ 9 8
-----------
2 3 4

Then how can A be sufficient?


The question asks "What is the sum of E, F, and G ?". In both valid cases (ii and iii) the sum of E, F, and G is 9. Check the solution it clearly explains this:

In the correctly worked addition problem above, A, B, C, D, E, F, and G are distinct digits. What is the sum of E, F, and G ?

AD
BD
CD
---
EFG

Notice that E can be only 1 or 2 (no sum of 3 two-digit numbers can give number more than 297).

(1) A, B, and C are consecutive odd integers. 3 cases are possible:

(i) A, B, and C are 1, 3, and 5 (it doesn't matter which is which) --> 1+3+5=9 then E (hundreds digit) can only be 1, which is not possible since we are told that the digits are distinct and we already have 1 (A, B, or C);

(ii) A, B, and C are 3, 5, and 7 (it doesn't matter which is which) --> 3+5+7=15 then E (hundreds digit) can only be 1. So, D can be 0, 2, 4, 6, 8, or 9. After trial and error we can get that only D=4 will give all distinct digits:
34
54
74
---
162

E+F+G=9.

(iii) A, B, and C are 5, 7, and 9 (it doesn't matter which is which) --> 5+7+9=21 then E (hundreds digit) can only be 2. So, D can be 1, 3, 4, 6, or 8. After trial and error we can get that only D=8 will give all distinct digits:
58
78
98
---
234

E+F+G=9.

So, as you can see in both valid cases (ii and iii) the sum of E, F, and G is 9. Sufficient.

(2) E = 2. After some trial and error you can find that several numbers can be found which will give different values for the sum of E, F, and G, for example: 58+78+98=234 and 38+78+98=214. Not sufficient.

Answer: A.
Manager
Manager
Joined: 07 May 2020
Posts: 67
Own Kudos [?]: 24 [0]
Given Kudos: 11
GMAT 1: 650 Q50 V28
GMAT 2: 720 Q50 V38 (Online)
Send PM
Re: In the correctly worked addition problem above, A, B, C, D [#permalink]
Bunuel wrote:
41396302717 wrote:
Hello Bunuel

I had a scenario like this

3 4
5 4
+ 7 4
------------
1 6 2

and the other can be

5 8
7 8
+ 9 8
-----------
2 3 4

Then how can A be sufficient?


The question asks "What is the sum of E, F, and G ?". In both valid cases (ii and iii) the sum of E, F, and G is 9. Check the solution it clearly explains this:

In the correctly worked addition problem above, A, B, C, D, E, F, and G are distinct digits. What is the sum of E, F, and G ?

AD
BD
CD
---
EFG

Notice that E can be only 1 or 2 (no sum of 3 two-digit numbers can give number more than 297).

(1) A, B, and C are consecutive odd integers. 3 cases are possible:

(i) A, B, and C are 1, 3, and 5 (it doesn't matter which is which) --> 1+3+5=9 then E (hundreds digit) can only be 1, which is not possible since we are told that the digits are distinct and we already have 1 (A, B, or C);

(ii) A, B, and C are 3, 5, and 7 (it doesn't matter which is which) --> 3+5+7=15 then E (hundreds digit) can only be 1. So, D can be 0, 2, 4, 6, 8, or 9. After trial and error we can get that only D=4 will give all distinct digits:
34
54
74
---
162

E+F+G=9.

(iii) A, B, and C are 5, 7, and 9 (it doesn't matter which is which) --> 5+7+9=21 then E (hundreds digit) can only be 2. So, D can be 1, 3, 4, 6, or 8. After trial and error we can get that only D=8 will give all distinct digits:
58
78
98
---
234

E+F+G=9.

So, as you can see in both valid cases (ii and iii) the sum of E, F, and G is 9. Sufficient.

(2) E = 2. After some trial and error you can find that several numbers can be found which will give different values for the sum of E, F, and G, for example: 58+78+98=234 and 38+78+98=214. Not sufficient.

Answer: A.

Sorry, I misread the question.
Director
Director
Joined: 29 Apr 2019
Status:Learning
Posts: 751
Own Kudos [?]: 583 [0]
Given Kudos: 49
Send PM
Re: In the correctly worked addition problem above, A, B, C, D [#permalink]
Correct option : A

we have :
1. AD + BD + CD = EFG (D unit is common for all Integer AD, BD, CD)
2. Correctly worked addition problem above, A, B, C, D, E, F, and G are distinct digits.
(Statement :1) A, B, and C are consecutive odd integers Eg. {1,3,5,7,9}
(Statement :2) E = 2

To Find : What is the sum of E, F, and G ?

Statement 1 :
Sum of two digit interger of 3 numbers, gives us 3 digit integer Number
D has to be same for all 3 integer in Unit place.
A, B, C has to be odd integer, keeping unit digit D constant
{11,31,51,71,91} are the set, and unit must give us 3 digit integer
easy : 91+71+51 = 213 = (E + F + G) = (2 + 1+ 3) = 6
Statement 1 sufficient , eliminates option D, and E

Statement 2 : E = 2, this cannot give us Solution but can be used as a check indicator,
Statement 2 is No Sufficieint - Eliminates B, and C
Tutor
Joined: 26 Jun 2014
Status:Mentor & Coach | GMAT Q51 | CAT 99.98
Posts: 452
Own Kudos [?]: 759 [1]
Given Kudos: 8
Send PM
Re: In the correctly worked addition problem above, A, B, C, D [#permalink]
1
Kudos
Expert Reply
carcass wrote:

In the correctly worked addition problem above, A, B, C, D, E, F, and G are distinct digits. What is the sum of E, F, and G ?
(1) A, B, and C are consecutive odd integers
(2) E = 2
Attachment:
challprobadd.jpg


We have:

AD
BD
+CD
------
EFG

We know that A, B, C are consecutive odd: 1,3,5 or 3,5,7 or 5,7,9

We can discard the set 1,3,5.
This is because the carry in this case would be only 1, i.e. E=1, which is not possible since all digits should be distinct.

Case 1: If we use 3,5,7: 3+5+7 = 15
So, there must be a carry from the units place else F=5 will match B=5
To have a carry from the units place, D must be either 4, 6, 8 or 9 (other digits either won't give a carry, or would be equal to one of A, B or C)

# With 4, we have:
34
54
+74
-----
162 => Satisfies all conditions. Thus, E+F+G = 1+6+2 = 9


# 6 cannot be used since the carry would be 1, resulting in F=6, which would be equal to D=6
# 8 cannot be used since the carry would be 2, resulting in F=7, which would be equal to C=7
# 8 cannot be used since we would have G=7, which would be equal to C=7

Case 2: If we use 5,7,9: 5+7+9 = 21
Possible values of D = 6, 7, 8, 9
Note: 1 is not possible since we would have D=F=1
2 is not possible since we would have D=E=2
3 is not possible since we would have G=C=9
4 is not possible since we would have G=E=2
5,7 or 9 is not possible since we would have D equal to one of A, B or C
6 is not possible since we would have F=E=2
8 is the only possible case:

58
78
+98
-----
234 => Satisfies all conditions. Thus, E+F+G = 2+3+4 = 9

Thus, Statement 1 is sufficient since we get a consistent value of E+F+G=9


Statement 2:
We only know that E = 2
There can be many possible scenarios for that:
For example: 97+87+67 = 251
97+87+47 = 231, etc.

Thus, it is not sufficient

Answer A
User avatar
Non-Human User
Joined: 09 Sep 2013
Posts: 32667
Own Kudos [?]: 821 [0]
Given Kudos: 0
Send PM
Re: In the correctly worked addition problem above, A, B, C, D [#permalink]
Hello from the GMAT Club BumpBot!

Thanks to another GMAT Club member, I have just discovered this valuable topic, yet it had no discussion for over a year. I am now bumping it up - doing my job. I think you may find it valuable (esp those replies with Kudos).

Want to see all other topics I dig out? Follow me (click follow button on profile). You will receive a summary of all topics I bump in your profile area as well as via email.
GMAT Club Bot
Re: In the correctly worked addition problem above, A, B, C, D [#permalink]
Moderator:
Math Expert
92912 posts

Powered by phpBB © phpBB Group | Emoji artwork provided by EmojiOne